0 Daumen
255 Aufrufe
Ich weiß überhaupt nicht wie ich diese Aufgabe lösen kann.......Kann mir vielleicht jemand helfen???Ich muss zeigen, dass der logische Schluss korrekt ist.
$$ \sum _{ j=1 }^{ k }{ \frac { { 3 }^{ 2(j-1) } }{ { 4 }^{ j } }  } =\quad \frac { 1 }{ 5 } ({ \frac { 9 }{ 4 }  })^{ k }-\frac { 1 }{ 5 } \quad für\quad ein\quad k\epsilon N\quad \Longrightarrow \quad \sum _{ j=1 }^{ k+1 }{ \frac { { 3 }^{ 2(j-1) } }{ { 4 }^{ j } }  } =\frac { 1 }{ 5 } ({ \frac { 9 }{ 4 }  })^{ k+1 }-\frac { 1 }{ 5 }  $$

Avatar von

1 Antwort

0 Daumen

Für die geometrische Reihe gilt
$$ \sum_{j=1}^k q^j=q\frac{q^k-1}{q-1}  $$
$$ \frac{3^{2(j-1)}}{4^j}=\frac{1}{9} \left( \frac{9}{4} \right)^j  $$
Also kann man \( q=\frac{9}{4} \) setzten
Daraus folgt
$$ \sum_{j=0}^k \frac{3^{2(j-1)}}{4^j}=\frac{1}{9}\frac{9}{4}\frac{\left( \frac{9}{4}\right)^k -1}{\frac{5}{4}}= \frac{1}{5}\left[ \left( \frac{9}{4}\right)^k -1  \right]$$

Avatar von 39 k

Ein anderes Problem?

Stell deine Frage

Ähnliche Fragen

Willkommen bei der Mathelounge! Stell deine Frage einfach und kostenlos

x
Made by a lovely community